Proving Convergence and Limit of a Sequence (Xn) in R

  • Thread starter Thread starter cb1020102022
  • Start date Start date
  • Tags Tags
    Limit Sequence
Click For Summary
The discussion focuses on proving that the sequence (An), defined as the average of the first n terms of a convergent sequence (Xn), is also convergent and has the same limit L. Participants suggest starting by writing out the first few terms of (An) to understand its behavior. The challenge lies in applying the definition of limits and finding an appropriate epsilon to show that the average approaches L as n increases. It is emphasized that while initial terms may not be close to L, later terms must be considered for convergence. The conversation highlights the need for a structured argument to demonstrate the convergence of (An).
cb1020102022
Messages
3
Reaction score
0

Homework Statement


let (Xn) be a sequence in R. Let (An) be a sequence defined as An=(X1 +X2+...Xn-1+Xn)/n. (Xn) is a convergent sequence and the limit of Xn as n goes to infinity is L. Prove (An) in convergent sequence and that the limit is also L.

Homework Equations





The Attempt at a Solution

 
Physics news on Phys.org
What have you tried? Where are you stuck.

Write out the first few terms of (An).
 
A1=X1, A2= (X1+X2)/2, A3=(X1+X2+X3)/3... I HAVE A THM IN MY BOOK THAT SAYS IF THE SUMMATION OF A SEQUENCE Xn CONVERGES THEN THE LIMIT OF Xn is 0. So I first assumed that the limit of Xn is not 0 the the numerator of An does not converge. But that does not really help. I am trying to figure out which convergence theorem to use...
 
So the n-th term, An, is the average of the first n terms of sequence (Xn).
 
right! But I am still stuck...
 
cb1020102022 said:
right! But I am still stuck...

You need to put together an argument using the definition of limit here. Pick an epsilon, how would find an N such that the average of the terms up to k is within epsilon of L for all k>N? The initial terms in the sequence may not be close to L, the later ones have to be. You need to take both groups into account.
 
Last edited:
Question: A clock's minute hand has length 4 and its hour hand has length 3. What is the distance between the tips at the moment when it is increasing most rapidly?(Putnam Exam Question) Answer: Making assumption that both the hands moves at constant angular velocities, the answer is ## \sqrt{7} .## But don't you think this assumption is somewhat doubtful and wrong?

Similar threads

  • · Replies 1 ·
Replies
1
Views
2K
  • · Replies 1 ·
Replies
1
Views
3K
  • · Replies 3 ·
Replies
3
Views
2K
Replies
4
Views
2K
  • · Replies 2 ·
Replies
2
Views
3K
  • · Replies 1 ·
Replies
1
Views
2K
  • · Replies 1 ·
Replies
1
Views
2K
Replies
1
Views
3K
  • · Replies 13 ·
Replies
13
Views
2K
  • · Replies 4 ·
Replies
4
Views
2K